Stay ahead of learning milestones! Enroll in a class over the summer!

G
Topic
First Poster
Last Poster
Brazilian Locus
kraDracsO   16
N 14 minutes ago by Giant_PT
Source: IberoAmerican, Day 2, P4
Let $B$ and $C$ be two fixed points in the plane. For each point $A$ of the plane, outside of the line $BC$, let $G$ be the barycenter of the triangle $ABC$. Determine the locus of points $A$ such that $\angle BAC + \angle BGC = 180^{\circ}$.

Note: The locus is the set of all points of the plane that satisfies the property.
16 replies
kraDracsO
Sep 9, 2023
Giant_PT
14 minutes ago
Disconnected Tree Subsets
AwesomeYRY   25
N 24 minutes ago by john0512
Source: TSTST 2021/5
Let $T$ be a tree on $n$ vertices with exactly $k$ leaves. Suppose that there exists a subset of at least $\frac{n+k-1}{2}$ vertices of $T$, no two of which are adjacent. Show that the longest path in $T$ contains an even number of edges. *

Vincent Huang
25 replies
AwesomeYRY
Dec 13, 2021
john0512
24 minutes ago
schur weighted
Ducksohappi   0
27 minutes ago
Schur-weighted:
let a,b,c be positive. Prove that:
$a^3+b^3+c^3+3abc\ge \sum ab\sqrt{a^2+b^2}$
0 replies
Ducksohappi
27 minutes ago
0 replies
Concurrency of tangent touchpoint lines on thales circles
MathMystic33   1
N 38 minutes ago by Giant_PT
Source: 2024 Macedonian Team Selection Test P4
Let $\triangle ABC$ be an acute scalene triangle. Denote by $k_A$ the circle with diameter $BC$, and let $B_A,C_A$ be the contact points of the tangents from $A$ to $k_A$, chosen so that $B$ and $B_A$ lie on opposite sides of $AC$ and $C$ and $C_A$ lie on opposite sides of $AB$. Similarly, let $k_B$ be the circle with diameter $CA$, with tangents from $B$ touching at $C_B,A_B$, and $k_C$ the circle with diameter $AB$, with tangents from $C$ touching at $A_C,B_C$.
Prove that the lines $B_AC_A, C_BA_B, A_CB_C$ are concurrent.
1 reply
MathMystic33
5 hours ago
Giant_PT
38 minutes ago
Balkan MO 2022/1 is reborn
Assassino9931   8
N an hour ago by Giant_PT
Source: Bulgaria EGMO TST 2023 Day 1, Problem 1
Let $ABC$ be a triangle with circumcircle $k$. The tangents at $A$ and $C$ intersect at $T$. The circumcircle of triangle $ABT$ intersects the line $CT$ at $X$ and $Y$ is the midpoint of $CX$. Prove that the lines $AX$ and $BY$ intersect on $k$.
8 replies
Assassino9931
Feb 7, 2023
Giant_PT
an hour ago
USAMO 1985 #2
Mrdavid445   6
N 3 hours ago by anticodon
Determine each real root of \[x^4-(2\cdot10^{10}+1)x^2-x+10^{20}+10^{10}-1=0\]correct to four decimal places.
6 replies
Mrdavid445
Jul 26, 2011
anticodon
3 hours ago
Inequality with rational function
MathMystic33   3
N 4 hours ago by ariopro1387
Source: Macedonian Mathematical Olympiad 2025 Problem 2
Let \( n > 2 \) be an integer, \( k > 1 \) a real number, and \( x_1, x_2, \ldots, x_n \) be positive real numbers such that \( x_1 \cdot x_2 \cdots x_n = 1 \). Prove that:

\[
\frac{1 + x_1^k}{1 + x_2} + \frac{1 + x_2^k}{1 + x_3} + \cdots + \frac{1 + x_n^k}{1 + x_1} \geq n.
\]
When does equality hold?
3 replies
MathMystic33
Yesterday at 5:42 PM
ariopro1387
4 hours ago
k A cyclic weighted inequality
MathMystic33   2
N 4 hours ago by grupyorum
Source: 2024 Macedonian Team Selection Test P2
Let $u,v,w$ be positive real numbers. Prove that there exists a cyclic permutation $(x,y,z)$ of $(u,v,w)$ such that for all positive real numbers $a,b,c$ the following holds:
\[
\frac{a}{x\,a + y\,b + z\,c}
\;+\;
\frac{b}{x\,b + y\,c + z\,a}
\;+\;
\frac{c}{x\,c + y\,a + z\,b}
\;\ge\;
\frac{3}{x + y + z}.
\]
2 replies
MathMystic33
5 hours ago
grupyorum
4 hours ago
k Perfect squares imply GCD is a perfect square
MathMystic33   1
N 4 hours ago by grupyorum
Source: 2024 Macedonian Team Selection Test P6
Let \(a,b\) be positive integers such that \(a+1\), \(b+1\), and \(ab\) are perfect squares. Prove that $\gcd(a,b)+1$ is also a perfect square.
1 reply
MathMystic33
4 hours ago
grupyorum
4 hours ago
Divisibility condition with primes
MathMystic33   1
N 4 hours ago by grupyorum
Source: 2024 Macedonian Team Selection Test P1
Let \(p,p_2,\dots,p_k\) be distinct primes and let \(a_2,a_3,\dots,a_k\) be nonnegative integers. Define
\[
m \;=\;
\frac12
\Bigl(\prod_{i=2}^k p_i^{a_i}\Bigr)
\Bigl(\prod_{i=1}^k(p_i+1)\;+\;\sum_{i=1}^k(p_i-1)\Bigr),
\]\[
n \;=\;
\frac12
\Bigl(\prod_{i=2}^k p_i^{a_i}\Bigr)
\Bigl(\prod_{i=1}^k(p_i+1)\;-\;\sum_{i=1}^k(p_i-1)\Bigr).
\]Prove that
\[
p^2-1 \;\bigm|\; p\,m \;-\; n.
\]
1 reply
MathMystic33
5 hours ago
grupyorum
4 hours ago
a